LSAT and Law School Admissions Forum

Get expert LSAT preparation and law school admissions advice from PowerScore Test Preparation.

 GLMDYP
  • Posts: 100
  • Joined: Aug 19, 2013
|
#11721
Hi! I find (B) is accurate as well. I find the right answer (E) to be too strong because there is a sense of absoluteness even though I admit that the stimulus has a sense of absoluteness as well.
Thanks!
 Ron Gore
PowerScore Staff
  • PowerScore Staff
  • Posts: 220
  • Joined: May 15, 2013
|
#11725
Hi, GLMDYP!

Your post suggests two questions: (1) why is answer choice (B) not a valid Must Be True inference; and (2) why isn't answer choice (E) too definitive to be correct?

You are correct that this is a Must Be True question. It contains both conditional reasoning and causal reasoning, though it is the conditional reasoning that permits an additive inference, which is tested in answer choice (E).

The facts in the stimulus are:

(1) without the intervention of conservationists, squirrel monkeys will become extinct;

(2) the squirrel monkeys will survive if large tracts of second-growth forest habitat are preserved for them; and

(3) squirrel monkeys flourish in second-growth forest because of the plentiful supply of their favorite insects and fruit

The third fact presents a causal relationship, regarding what causes the squirrel monkeys to flourish in the second growth forests. This relationship is not tested, though it could have been.

What answer choice (E) does test is the inference permitted by the combination of facts (1) and (2), expressed below as two conditional relationships:

..... (1) squirrel monkeys extinct ..... :arrow: ..... intervention conservationists

..... (2) large tracts preserved ..... :arrow: ..... squirrel monkeys extinct

..... inference: ..... large tracts preserved :arrow: intervention conservationists

Answer choice (E) contains the inference expressed immediately above. Because it is a valid conditional reasoning inference, it is a proper Must Be True correct answer choice.

While answer choice (E) presents an absolute relationship, it is conditional. Answer choice (B) is not conditional, but is a definitive statement regarding what will occur. Even though answer choice (B) appears to be a more cautious inference from the stimulus, in that it says "at least some," it is a future prediction about what "will" occur.

However, the facts in the stimulus concerning conservationists and the preservation of the second growth forests were conditional statements, with no facts regarding what will, in fact, occur. That is why answer choice (B) is incorrect.

Please let me know if you have further questions, or if I can assist you further.

Thanks,

Ron
 GLMDYP
  • Posts: 100
  • Joined: Aug 19, 2013
|
#11728
This is very helpful Ron! Thank you!
 srcline@noctrl.edu
  • Posts: 243
  • Joined: Oct 16, 2015
|
#27620
Hello Ron,

I think I'm getting a bit lost on how to diagram this. I diagrammed this as :

1. w/o the intervention of conservationists :arrow: squirrel monkeys will become extinct.
Im confused on how to diagram this , but the contrapostive would say something like squirrel monkeys will not become extinct with the intervention of conservationists.

2. if large tracts of second-growth forests habitat are preserved for them :arrow: will survive.
(+) If Squirrel monkeys will not survive :arrow: large tracts of second growth forest habitat are not preserved for them

Not sure how to combine these to lead to answer choice E.

Thankyou
Sarah
User avatar
 Jonathan Evans
PowerScore Staff
  • PowerScore Staff
  • Posts: 726
  • Joined: Jun 09, 2016
|
#27651
Sarah, good job here. Difficult question. You've more or less got it right. Let's get explicit:

no intervention :arrow: extinct monkeys

extant monkeys :arrow: intervention

preserve forest :arrow: extant monkeys

extinct monkeys :arrow: no forest

Always read from left to right. Let's see where we can go here. No intervention leads us to no forest. Preserve forest leads us to intervention. That's it. Those are the only connections we can make.

(B) is an attractive wrong answer because it has nice weak language (for a Must Be True) but flubs the conditionals.

(E) says no intervention. Where can we go from there? We can arrive at no forest. We got that. Excellent. We found the credited response.
 srcline@noctrl.edu
  • Posts: 243
  • Joined: Oct 16, 2015
|
#27687
Hello Jonathan,

Wow, okay I see the chain now b/c of the necessary condition of extinct monkeys.

Thank you for your explanations.

Sarah
 jlam061695
  • Posts: 62
  • Joined: Sep 17, 2016
|
#32317
I am having trouble with the word "without" in this stimulus. How would the diagram be drawn? And why would E be the correct answer instead of C? This is how I diagrammed the stimulus:

Ex (squirrel monkeys will become extinct) :arrow: IC(without the intervention of conservationists)
PS (plentiful supply of insects and fruits) :arrow: LT2ndP (large tracts of second-growth forests are preserved) :arrow: Ex (squirrel monkeys will not become extinct/they will survive)

To me, C is the correct answer because it says that squirrel monkeys will become extinct without plentiful supplies of insects and fruit, which would be diagrammed: Ex :arrow: PS. This is the contrapositive of what I have in the stimulus.
 Adam Tyson
PowerScore Staff
  • PowerScore Staff
  • Posts: 5153
  • Joined: Apr 14, 2011
|
#32344
Hey jlam, let me see if I can help.

First, dealing with "without" (which is a special Necessary Condition indicator, along with "except", "unless", and "until"): when you see these words, using our methodology you would treat whatever they are attached to as a necessary condition. Here, they are attached to "intervention of conservationists", so that tells us that "IC" is the necessary condition. The other thing in the relationship must be negated to form your Sufficient Condition. Here, that other thing is "squirrel monkeys will become extinct" (which I will just call EM, for extinct monkeys). Negated, then, we get EM as the sufficient condition for IC: EM :arrow: IC (read that as "if the monkeys aren't extinct, conservationists must have intervened). Looks like what you diagrammed is a Mistaken Negation of what it really said. For help on those special indicators, check out your course materials or your LR Bible for the chapter on Conditional Reasoning. If you are working without our books, that's okay too - we have a lot of discussions both here and in the blog that talk about how to do them, and you want to look into what we call the "Unless Equation"tm.

The second issue you are having is with the inclusion of the Plentiful Supply idea in the stimulus. You've treated that as a condition sufficient to prove that we are looking at a large tract of second growth forest, but it is not only not sufficient for that, it isn't in a conditional relationship at all! As Ron described in his prior posts in this thread, that is purely a causal argument, and it isn't tied to our conditional claims in any way or used to make a conclusion. For all we know, there is a similar plentiful supply of insects and fruit at my neighborhood Whole Foods! Leave that claim out of the conditional chain, because it is not conditional and not tied to either EM or IC.

What we do know about those forests is that if they are preserved (let's call that PF), then the monkeys will not become extinct (EM). Now we can make our conditional chain:

PF :arrow: EM :arrow: IC

and the contrapositive:

IC :arrow: EM :arrow: PF

Either of those should allow you to select E with confidence, and reject C because it tries to tie in the causal claim to the conditional chain, where it has no business.

I hope that clears it up a bit!
 jlam061695
  • Posts: 62
  • Joined: Sep 17, 2016
|
#32398
Thanks much for the explanation, Adam! That helped a lot!
 fmihalic1477
  • Posts: 27
  • Joined: Jan 09, 2017
|
#34829
Hi all,

This morning I was making copies of sections that I will use in the future as experimental sections and in passing came across a #23/must be true in a logical reasoning section. I thought "I bet this is fun", so I quickly did the problem with no pencil in hand to diagram (mistake #1).

Unfortunately, I did the problem so quickly and did not note the LSAT nor the section. I remember that it was about some type of monkey and whether or not they could survive with the help of the conservationists and this particular type of fruit that they love so much. I understand this is like showing up to a party empty handed, but I'm sorry!

To my mistake..

The stimulus read something like : "Without the intervention of the conservationists, the monkeys will surely go extinct. However, they can survive if they have an abundant amount of x berry. X berry is so valuable to the monkeys because it provides them with great nutrition."

Which of the following can properly be inferred from the statements above, if true?

Immediately I know this answer revolves around those two conditional statements.

E - - Intervention (conversationalists)
Intervention - - E

AAX - - Can survive
Can survive - - AAX

E was the correct choice.

E was something along the lines of "The conservationists will have to be able to provide enough of berry x for the monkeys". It would really help if I knew the exact problem. Hopefully the amount of detail I've given will make it easy to find.

In conclusion, if the monkeys are to survive, they will have both the intervention by the conservationists and enough of berry x.

Do I have this right?

Thank you for any help you can provide,

Frank

Get the most out of your LSAT Prep Plus subscription.

Analyze and track your performance with our Testing and Analytics Package.